Laplace Transform question with unit step function

Click For Summary
The discussion revolves around a Laplace Transform problem involving a unit step function. The initial solution provided includes a function f(t) and its corresponding Laplace Transform, but there is uncertainty regarding the correctness of the fifth term in f(t). Feedback suggests that while the first four terms are correct, the fifth term needs revision, and emphasizes the importance of verifying each part of the solution. The conversation highlights the need for careful analysis of the function across different intervals of t. Clarifying these points will help determine the accurate solution to the problem.
jamesdocherty
Messages
14
Reaction score
0

Homework Statement



upload_2015-5-7_19-12-23.png


Homework Equations


Laplace Trasformations

The Attempt at a Solution


a. done
b. f(t)= t -3*t*u(t-1) + 4*u(t-1) -3*u(t-2) -2*t*(t-2)
c. 1/(s^2) - (3e^-s -2e^-2s)/(s^3) + (4e^-s -3e^-2s)/s
d. 1/(s-1) * (1/(s^2) - (3e^-s -2e^-2s)/(s^3) + (4e^-s -3e^-2s)/s)

These are the answers i got but my friend got all different ones and we are not sure who is correct. Thanks for any help.
 
Physics news on Phys.org
jamesdocherty said:

Homework Statement



View attachment 83171

Homework Equations


Laplace Trasformations

The Attempt at a Solution


a. done
b. f(t)= t -3*t*u(t-1) + 4*u(t-1) -3*u(t-2) -2*t*(t-2)
c. 1/(s^2) - (3e^-s -2e^-2s)/(s^3) + (4e^-s -3e^-2s)/s
d. 1/(s-1) * (1/(s^2) - (3e^-s -2e^-2s)/(s^3) + (4e^-s -3e^-2s)/s)

These are the answers i got but my friend got all different ones and we are not sure who is correct. Thanks for any help.

The first 4 terms on the right of (b) are OK but the fifth term is incorrect. You should develop the habit of always checking these things for yourself; just look at f(t) and your right-hand-side for 0 < t < 1, then for 1 < t < 2, then for t > 2. What does your answer in (b) look like for t > 2?
 
Question: A clock's minute hand has length 4 and its hour hand has length 3. What is the distance between the tips at the moment when it is increasing most rapidly?(Putnam Exam Question) Answer: Making assumption that both the hands moves at constant angular velocities, the answer is ## \sqrt{7} .## But don't you think this assumption is somewhat doubtful and wrong?

Similar threads

  • · Replies 6 ·
Replies
6
Views
1K
  • · Replies 1 ·
Replies
1
Views
1K
  • · Replies 7 ·
Replies
7
Views
2K
  • · Replies 9 ·
Replies
9
Views
3K
  • · Replies 10 ·
Replies
10
Views
2K
  • · Replies 4 ·
Replies
4
Views
3K
  • · Replies 8 ·
Replies
8
Views
2K
  • · Replies 3 ·
Replies
3
Views
3K
Replies
2
Views
2K
  • · Replies 1 ·
Replies
1
Views
941